You are on page 1of 154

1.

Let be real numbers, each less than one, satisfy

(I) Let be the smallest integer such that

Show that all the sums are


positive.

(II) show that there exists two integers and , such that the numbers

are all positive.


Ans:
PART 1: clearly since is
the smallest integer with so

and etc.this way


all sums are positive.

PART 2 : for convenience i replace by

Let us consider the maximum among the sums

[ and so is not the maximum]


Let that maximum sum be
now by our assumption implying
implying

this way are all positive.

replacing by we get the result.

2. A troop long starts marching. A soldier at the end of the file steps
out and immediately turns around and marches back at the same speed. As
soon as he reaches the end of the file, the troop stops marching, and it is
found that the troop has moved by exactly What distance has the
soldier traveled?
Ans: Let x and y be the speed (measured in meter per second) of the troop
and the soldier passing the troop respectively. The time it takes the soldier
passing the column is seconds and on the way back to the end of the column
he spends seconds. The column has moved 5 meters in seconds while the
soldier has marched from the tail to the head and back to the tail of the
column. In other words

or

or solving
now the soldier had travelled at speed for time.

HENCE distance covered by him

3. Let be an interior point of the trinagle .assume that


meet the opposite sides of are respectively show that
Ans: BY Ceva's Theorem

or,

IN TRIANGLE APE,

IN Tr. ECB

IN Tr. PBD,

Using 3,4,5, in 2

4. Inside equilateral triangla an arbitrary point is taken from which


perpendiculars are dropped on respectively. P.T.
the ratio
is unique irrespective of P and find its
value.
Ans: By Carnot's theorem:

, where we get
5. is a triangle right angled at , be any point on side . drawn
parallel to meets at . is the foot of perpendicular from to .

If show that

.
Ans:
6. is a chord of circle
(a) find a point on the circumference such that is maximum
(b)find a point on the circumference which maximizes .
Ans: let subtend angle in centre of circle, let

let where =angle subtended by , = angle


subtended by
so
differentiating above w.r.t and equating to we get second
derivative is always negative
so clearly is mid point of arc

Second part is similar.

7. There are doors , ,....., and people , , ,....,


.Initially all doors were closed.Person goes in and opens all doors. goes
in and opens all , ,..., ...and so on... goes in and opens , ,.....
...leaving all other doors untouched.Finally goes in and opens
if it was closed and closes if it was open.At the end,how many doors remain
open?
Ans: no. of doors whose state has changed an odd number of times=no. of
positive integers between and which have odd number of divisors=no.
of perfect squares between and = .Thus, is the final
answer.
[ if be a positive integer and its prime factorisation be

where 's are the prime factors of and 's are their respective powers then
number of divisors of is

if is square then 's are even so is odd, hence has odd number of
divisors.

Conversely if has odd number of divisors, must be odd, so each of


must be odd so each of the 's should be even. So is a
square.]

8. Suppose there are k teams in a round robin tournament,each team plays


against all other teams and no game ends in a draw.Suppose that the ith team
loses games and wins games.Show that:-

Ans: Since team i plays games and loses games,and there can be
no draw,we have ...now we are to prove that

Now becomes

..now in a round robin league,each

match is counted as a win and a loss both..so =total number of matches

played= = ...thus cancelling out,we have .


9. Suppose that is a polynomial of degree such that,

for

Find the value of .


Ans: Let .

Then the polynomial vanishes for that is,

To find we set and get,

thus,

And ,

for odd

for even .

10. If is a positive integer greater than ,such that is a perfect


square,then show that is the sum of three perfect squares.
Ans: Let or since is integer, either
divides or

CASE 1: divides i.e,

then or

CASE 2: divides ,

then

hence proved.

11. Show that the number with digits is divisible by .


Ans: we assume that is divisible by so
[why?]

so [for some integral r]

hence which is divisible by

is divisible by by induction

12. Let be numbers such that each is either or If


prove that divides .
Ans: Let the sum be denoted by . Initially which is divisible by .

Suppose we change the sign of any one of 's,(i.e, we make it positive if it


was negative or vice-versa). Since occurs in exactly four of the terms, we
consider the following cases:

a)All four terms were negative(i.e, were equal to by question)


Then these terms added to . But sign chinge of the makes sum of these
terms .
So new sum

b)Three were positive, one negative


In a similar fashion, new sum=

c)Two positive, two negative:


New sum=

d)One negative, three positive:


New sum=

e)All positive:
New sum=

We observe from the above cases, that after the change of signs, new sum
remains divisible by (that is, the sum modulo in invariant)
We keep on changing signs of different 's until all terms become and the
sum equals .
But the sum will still be divisible by

So divides .
13. Suppose and are positive integers greater than or equal to .Show that
is divisible by .
Ans: And here is my solution:-
We have:-

= since cancelling out terms from


gives .There are n terms on .Now,we multiply and divide by

.This gives:- = ,where is a


positive integer since the binomial coefficient is a positive integer.Thus
...which implies that is divisible by .

14. For , show that where is a positive


integer.
Ans: applying AM-GM inequality to the quantities

or

or

or .
15. consider the circle whose equation is and the
parabola with the equation find the minimum value of the length of
the segment as moves on the circle and moves on the parabola .
Ans: Any point on is .

Centre of is . Distance between is

Let . is minimum if is minimum.

So differentiating, , equating to we get


.

Now so is minimum at

So is so

So Required shortest distance = radius of circle= .

16. Given and are two quadratic polynomials with rational coefficients.
Suppose and have a common irrational solution.
Prove that for all where is a rational number.
Ans: Irrational roots occur in conjugate pairs.......now as the degree of both
the polynomials is 2 so if they have one common root then both the irrational
roots of the polynomial are equal.........now let and
......as both the polynomials have rational coefficients
so and both are rationals......
.now a rational number r[since are rationals]

Hence...... where is a rational no.

17. Consider the parabola . Let be any point inside it, so that
. Let be the focus of the parabola.
Find the point on the parabola so that is minimum. Also show
normal to parabola bisects angle .
Ans: Point and parabola with focus .
Point on this parabola.

.
.
has a minimum if .

Solving: .

Point .
x-axis

Let be the tangent to the parabola at intersecting at . Let


be the normal intersecting at .(draw figure)
Clearly so and
also
We see that implies so slope of normal at is

or

so also slope of

Angle bteween and is


so
so bisects .

18. If are positive numbers, show that

Ans: We have or or

so

applying similar inequalities for pairs and and adding and simplifying
we get

.
19. Let be positive reals such that
Prove that .
Ans: Applying AM-GM on and we get

similarly , ,

Multiplying and using we get

20. Let be the ellipse with centre at origin whose major and minior axes
are of length and respectively.
Let be the acute angle at which is cut with the circle with centre at
origin(i.e. is the acute angle of intersection of their tangents at the point of

intersection). Prove that maximum value of is .


Ans: equation of tangent of ellipse at is with slope
(say)

tangent of the circle is of form (assuming radius of circle=


) with slope (say)

So
for to be maximum should be maximum. after rearrangement

let this be eq. 1


above is maximum if its denominator is minimum.

By AM-GM, or
putting this minimum value in eq. 1 we get maximum of is

hence maximum value of is .

21.show that if then >


Ans: Proceed by induction
True for
Let's assume its true for
To show,
Now,

To show,

Consider the function

Now
So

Hence

Hence is decreasing, so

Hence we're done.


22. Let be any values in the closed interal .Show that
.
What is the maximum value attainable by F and at what values of
is the maximum value attained?
Ans: First part.
Pick up one pair of sin and cosine from the function .
Let us have as one pair.Now when we expand this we
get

now using AM-GM inequality we may write ,

So,each pair of is less than equal to .

Hence product of such pairs is less than equal to .


Second part.

From the first we can say that the maximum value attainable by F is .
So thus we have to find out for which values of ,

now from here using some trigonometric equation we can say that or
as .

23. Let be a positive number. A sequence { } of real numbers is defined


as follows:

and in general,

for all .

(a)Show that , for all ,


(b)Hence find .
Ans: First part

Induction on
See for it is obvious.
Let for some

Then we would show it true for


then take =
and define and
Then since is same as except that in the of has been replaced by
so it obeys the condition which we need to prove (From Induction
hypothesis)
Then we are precisely done!!
Part 2

See is a monotonically decreasing sequence since and it is bounded


(below by ) so limit exists.
Now let be the limit.
Then

24. For real numbers show that

.
Ans: For quantities and applying triangle inequality,
we get

or

Obtaining two similar inequalities for and


and adding the three inequalites thus obtained we get

25. Show that number of ways in which four distinct integers can be chosen

from , such that no two are consecutive is .


Ans: Let us assume the four integers be since are not
consecutive we have
Now may or may not be consecutive. We can choose
them from terms.

So Total number of ways to choose = .

26. Show that if then > .


Ans: Proceed by induction
True for
Let's assume its true for
To show,
Now,

To show,

Consider the function

Now
So

Hence

Hence is decreasing, so

Hence we're done.


27.Let be an infinite sequence of circles lying in the positive quadrant
of the -plane, with strictly decreasing radii and satisfying the following
conditions. Each touches both the -axis and the -axis. Further, for all
, the circle touches the circle externally. If has radius cm,

then show that the sum of the areas of all these circles is sq cm.
Ans: Let is the radius of the circle From all the given tangency
conditions we have

[Say]
Hence the required sum,

[Since
]

sq cm. [Putting values of and ]

28. Suppose that the equations , and


only have positive roots. Show that .
Ans: The equations have positve(hence real) roots. So their discriminants are
positive or zero.

So, or similarly ,

Multiplying the inequalities we get, This is possible iff


, ,

from the first two equalities or


Given roots of equation are positive hence sum of roots are positive, So ,

, are positive which is possible iff are of same sign.

So cannont equal . So Similarly,

Hence .

29. Let and be positive integer such that is a


prime. Then show that is also a prime number.
Ans: Let then,

As both the factors are hence contradiction.

has to be a prime.

30. Find the maximum and minimum values of in the


closed interval .
Ans:
so
Clearly since and in so or increases
in

So is maximum at and minimum at

So, required maximum value=

Minimum value=

31. Suppose is a prime number such that and are also primes.
Show that .
Ans: This is a pretty simple problem, clearly for some ...

now by the given condition, and are both primes, implying that
both and are primes. So the problem basically reduces to finding all
primes such that , , are all primes.

For ,clearly this doesn't hold. Now for any ,we must have,
or .In the first case, we have ,so it cannot be
prime, and in second case, ,another contradiction. So, is the only
solution, which gives as the only solution.
32. Prove that for any odd integer when divided by always leaves
remainder
Hence or otherwise show that we cannot find integers such
that

Ans: For any odd integer we have,

Suppose
For even integer
Hence for all integer we have
Hence But
CONTRADICTION!

33. Show that there is one value of for which .


Ans: Let

Clearly and is continuous in interval . So by


intermediate value theorem
takes all values between and . So assumes the value too.

So there is a real root of in the interval .

Hence Proved.

34. A vessel contains gallons of wine and another gallons of water. From
each vessel, gallons are taken and transferred to the other. From resulting
mixture, gallons are again taken out and transferred into the other. After
second transfer, quantity of wine in each vessel remains equal as it was after
first transfer prove that .
Ans: Let vessel contain wine and vessel contain water initially..

After first transfer,


vessel contains gallons wine.
vessel contains gallons wine.

So, fraction of wine in


fraction of wine in

During second transfer, quantity of wine pulled out from:-

Now from , gallon is pulled out and the wine which is pulled from ,i.e,
gallon, is poured in , and resulting mixture contain same quantity of
wine as in first transfer,i.e.,

Simplifying we get .

35. If a circle intersects the hyperbola at four distinct points ,


Prove that .
Ans: Let the circle be,

Since it intersects the hyperbola at four points, it passes through


So or

Clearly product of roots of above equation [By, Vieta's Theorem]

Since it passes through


are roots of the equation.

So or

so .

36. Show that there is no real constant such that


for all , being real.
Ans: Let there be a such that,

or [ ]

or or

RHS of above is independent of iff which contradicts . For all


other , depends on .

Hence, there is no real constant such that for all


, being real.
37. red balls, black balls, white balls are to be arranged in a row.
Suppose two balls are of same color and are indistinguishable. Given
prove that number of possible arrangements is maximum
when .
Ans: Number of ways where things can be arranged with of one
type, of second, of third is

Now being constant, is maximum if is minimum,


which happens at according to question.

Let us assume the contrary, there are not all equal to for which

Now three cases may arise WLOG,


i) ii) iii)

Let us consider 1st case. for positive


integers
Clearly

Then
which implies

But this is contradictory since, each term of LHS of above>each terms of


RHS of above
and the terms are positive.

So for all

Similar contradiction can be shown in other two cases.

So possible arrangement is maximum when .


38. Consider the equations in and :
, where belongs to set of reals.

Show that these equations admit real solutions in and .


Ans:

implies

Also , substituting value of from we get,

Simplifying,

Above is a cubic equation and will be satisfied for at least one real .

If is real, is also real.

Hence, The equations have real solutions.

39. Suppose { } with the following addition and multiplication rules,

A system of polynomials is defined with coefficients in . The sum and


product of two polynomials in the system are usual sum and
product,respectively, where for the addition and multiplication of coefficients
the above mentioned rules apply. For example, in the system,

Show that in the system ( ) is not factorizable ,that is, one cannot
write

Where and are elements of .


Ans: Suppose,

Now, comparing the coefficients,

....(1)

........................(2)

........................(3)

....(4)

Now, from (2),

Hence, but we have

Hence, contradiction!!!
40. Consider the squares of an chessboard filled with the numbers 1 to
64(see the figure below). If we choose squares with the property that there
is exactly one from each row and exactly one from each column, and add up
the numbers is the chosen squares ,show that the sum obtained is always

Ans: Basically we are choosing the numbers

Where belongs to the set { }

Hence,
41. Let be a real valued function of variable such that takes both
positive and negative values and .Show that there is a real
number such that is an increasing function .
Ans: Since , so is strictly increasing.

Now assumes both positive and negative values, hence there is exactly
one root of , say (since strictly increases)

Now is increasing hence when

Hence is an increasing function when .

42. Let be numbers such that each is either or If

.
Ans: Let the sum be denoted by . Initially which is divisible by .

Suppose we change the sign of any one of 's,(i.e, we make it positive if it


was negative or vice-versa). Since occurs in exactly four of the terms, we
consider the following cases:

a)All four terms were negative(i.e, were equal to by question)


Then these terms added to . But sign chinge of the makes sum of these
terms .
So new sum

b)Three were positive, one negative


In a similar fashion, new sum=

c)Two positive, two negative:


New sum=

d)One negative, three positive:


New sum=

e)All positive:
New sum=

We observe from the above cases, that after the change of signs, new sum
remains diviisible by (that is, the sum modulo in invariant)
We keep on changing signs of different 's until all terms become and the
sum equals .
But the sum will still be divisible by

So divides .

43. At time , a particle is at point on real line.At time , the particle divides
to two and instantaneously after division, one particle moves one unit to the
left and other moves one unit to the right. When two particles meet, they
destroy each other leaving nothing behind. how many particles will be there
after time ?.
Ans: With smaller examples say at times and , the particle number is
trivially and at times and , it is .

Suppose we make hypothesis that there are particles at positions on


number line after time=
Then consider the point on the number line as origin. In next time the
particle at will move the same way as it had moved in its previous time,
hence become particles one of which will be situated at unit right other
at unit left from the point on number line. Same will happen for the
particle at

But for the particle at , the position at units left from it is the origin.
Same is true for the position of units right for the particle at . Hence
among the total of four particles as described above, two will destroy.

So after time also particles will be so our hypothesis was


correct.

So at time these two particles divide into two each, resulting in four
particles.

So number of particles after time = .

44. or natural number let . If is the g.c.d of and prove


that divides .
Ans:

So
Hence divides 81.

45. Using calculus, sketch the graph of following function on plain paper.

.
Ans: Clearly

i) cuts at and at ( ).

ii) implying it is even function and symmtric about

iii) It has infinite discontinuity at

iv) which has root at

v) is positive in right and negative in left of So increases at


, decreases at , minimum at

vi) Clearly at and other


wise. So is concave up at concave down otherwise.

vii) Let then Since it is a degree two


equation hence have at most two asymptotes.

viii)equating Coefficient of to we get so so those are the


only asymptotes.
Using above conditions we trace the curve.
46. Let, be polynomials of x, each having all integer
coefficients, such that

Assume that is not the zero polynomial. Show that and


.
Ans:

But a polynomial can never be bounded,

Because after a certain time [after Max(/min){ } ,where 's are the root of a
polynomial] polynomial becomes monotonic, unless,

1.) Polynomial is constant

2.) Polynomial becomes constant after a certain point

But case 2 is not possible, because in that case has


infinitely many roots,hence becomes an identity [for all
x]

Hence,

But, has integer coefficients,

[ is not a zero polynomial]


47. Let , where , , , are integers. The sums

of the pairs of roots of are given by . Find .


Ans: Let the roots of be

Wlog,

Now, the largest sum is

and second largest sum is

And, smallest sum is

Seccond smallest sum is

But we can't say anything about ( ) and ( )

So, two cases arise,

Case-1:

Solving these we get,


But this case is not possible as [not an integer]

Case 2:

Solving these we get,

And The polynomial becomes,

48. Let be a point and be . If be slope of


find .

Ans: The required slope

So

[by L hospital rule]

.
49. Show that for all

Ans: Let

which is true.

Let be true

Now

which is true.

50. Let, wherew and . Prove that the function


satisfies the following property:

{ }

for all not equal to and for all .


Ans: Denote by and by Hence

Without loss of generality we assume


[AM-GM]

51. The sum of squares of the digits of a three-digit positive number is


, while the sum of the two digits in the unit’s and the ten’s place is times the
digit in the hundred’s place. Further, when the number is written in the
reverse order, it is increased by .Find the number.
Ans: Let is unit’s digit, is ten’s digit and is the hundred’s digit.

Then from the given information we get

… (i)

…(ii)

…(iii)

From (iii) we get

Now from (ii) we get


So is a multiple of , can be .

For

not possible as

then

Similarly for

& are not possible.

For

So the only solution is .

52. Consider the equation , where and are complex


numbers. Suppose that this equation has a pair of complex conjugate roots.
Show that both and are real.
Ans: Let three roots of the equation

are [ Let are complex conjugates]

Now … (i)

… (ii)
… (iii)

From (ii) we get

[ are complex conjugates so they are real]

Now as

… (iv)

are complex conjugates so … (v)

From (iv) & (v) we get

[from (iii)]

Now is real and is real

so

53. Describe the set of all real numbers which satisfy

Ans:
Hence we have,

Therefore from we get,


But Hence or in other words,

54. Suppose that all roots of the polynomial equation


are positive real numbers. Show that all the roots
of the polynomial are equal.
Ans: Let us assume the roots to be
Clearly we have for all
Also from the polynomial we have,
and
Since 's are positive applying on 's we have,

Evaluating, we get the equality case, when Hence all 's are
equal.

55. Let J={0,1,2,3,4}.For x,y in J define to be the remainder of the


usual sum of x and y after division by 5 and to be the remainder of the
usual product of x and y after division by 5 .For example while
.Find x and y in J,satisfying the following equations
simuntaneously:

.
Ans: Using directly the standard product and sum signs and considering we
are in , the system is :
Adding the two lines gives and multiplying the first line by two gives
then

Hence the answer : .

56. Show that every circle passing through and is orthogonal to


the circle .
Ans: Let equation of the family of circles through an be

It passes through and

solving whatever be

Comparing with general equation of circle


we get,

Condition for orthogonality is

Putting respective values separately in LHS and RHS of expression (i) we


see that condition for orthogonality is satisfied.

HENCE: Every circle passing through and is orthogonal to the


circle .
57. Suppose are positive real numbers satisfying

for

Find . Also show that .


Ans: Given,

[dividing by ]

Hence,

......................
..............................

adding,

So

or .

58. Consider the set of points


{ are non-negative integers }

Find the number of squares that can be formed with vertices belonging to
and sides parallel to coordinate axes.
Ans: Let us start with the squares of sides square unit.

Least possible value of top right corner coordinate is Shifting


continuously to right from , possible values of are ,
. Moving continuously upwards from possiblt points are
so there are squares of size sq. units

For sq unit squares, starting from we have on right,


and upwards. So there are squares of sq.units

Similarly for sq unit squares, they are respectively


in number.

So total squares =

59. Prove that,

Ans: Let

So

or
Let I

or I

Now, in

So I

or

or .

60. i) If and are positive integers such that divides , show that for every
poitive integer , and are relatively prime.

ii) Consider the smallest number in each of subsets (of size ) of


. Show that arithmetic mean of the numbers thus obtained is

.
Ans: i)

Let (since divides )

So
The prime factors of and are same. Primes dividing divides as
well. Hence no primes dividing divides

Hence

So, and are relatively prime.

ii) Consider { }, { }. Consider a graph with white


vertices as element subsets of and black vertices with element
subsets of .
We join a black vertex and a white vertex iff by removing the lowest
element from the subset represented by we get the elements of . For e.g
if { } and we join them.

clearly degree(number of lines originating from a vertex) of each white


vertex is its lowest element, e.g. { } implies it is joined to {
} and { } so degree of which is its smallest element.
So sum of smallest elements

is the total edges(number of line segments) of which is .

again sum of smallest elements =

equating implying

.
61. Find all possible real numbers which satisfy the following set of
equations:

Ans: Without loss of generality, we may assume that is the greatest among
the five given numbers. Then ,and hence
.
Also, , and since , we get .
Hence also , and - giving .
Also, since , we get , and finally we also get . So, we need to
solve which gives the following solutions:
, and .

62. If are positive real numbers such that prove that,

.
Ans: Let

is minimum iff both and are i.e.

But . So

So minimum value of P is

So .
63. Suppose is a function such that and is continuous for
every real number . If for all ,then show that

for all .
Ans: We have,

64. Study the darivative of the function

for ,
and roughly sketch the graph of on plain paper.
Ans:

, ,
Clearly Clearly for and
for

So is maximum at . It increases at left of and decreases at right


of

Maximum of =

Clearly for all . Hence it is concave down.

Using above information we trace the curve.

65. The circles with radii respectively touch each other


externally. Centres of lie on . touches them from top. Find
ordinate of centre of circle that lies in region enclosed by and
touches all of them.
Ans:
WLOG, assume pass through origin since only y-coordinate is needed.
So centres of are and respectively.

Let centres of be and centre of reqired circle be with


radius .

touch hence,

Solving, centre of is
touches

, ,

Solving we get [Since touches from top, we take positive value of


]

So required ordinate=2/3.

66. If and are the lengths of the sides of a triangle and if and
are the lengths of the perpendiculars drawn from the circumcentre onto the
sides and respectively, then show that

Ans: From geometry, we have,


and where is the circumradius of and
is
Hence we have,

Similarly we have,

and
For the angles and of a triangle we have,
67. Show that there is exactly one value of which satisfies the equation

.
Ans: we know that

Also [By AM-GM]

So iff and holds.

Now iff or so

Also satisfies so exactly one value of satisfies

which is .

68. If , then show that,

.
Ans:

put

[integration by parts]
by evaluating the limits and we get,
69. Let be a point on a straight line segment such that
. Let be a point on the circle with centre at and radius such that
.(see figure). Show that .
Ans:

Ans: Refer to the

We are given following information:

From (1) we easily infer [Since


radius of the circle we r working with]
is the internal bisector of ; let

And we also observe readily that is isosceles.

Again using the given fact

So,we are having and in which

so now corresponding angles of the similar will be equal.

the opposite of in the opposite of in


Hence readily we get .

70. Evaluate:

.
Ans:

71. Evaluate:

.
Ans: ,
72. Show that for all real , the expression ( where are real

constants with ) has minimum value . Also find the value of


for which the minimum value is attained.
Ans: Let

Now equating we have . Also is always positive[


given]

So is minimum at .
The minimum value is .

73. If , then show that,

.
Ans: We have

where
which simplifies off to

Thus where

Therefore required expression .

74. Consider the circle of radius with its centre at the point . From this
initial position, the circle is rolled along the positive -axis without slipping.
Find the locus of point on the circumference of the circle which is at the
origin at initial position of circle.
Ans: Let the linear speed of the circle be , angular speed be , time and
radius (I know radius is given, but still...). Further let be the angle
subtended at the center by the point at time . Since the point is originally at
the origin, hence initial phase . and denotes the co-ordinates of the
point at any time

Now, due to the motion of the co-ordinate of the point, we have


. Since the circle rolls without slipping, we have and
hence
Now, coming to the y- coordinate, see that the co-ordinate can be
represented as a function of , viz, .(Draw the diagram at any
time and try to see why this equation holds ....)
Now,
Thus we have
Now here , so we have and which is the
required locus of the point.

75. Let and be two positive integers. Define for using the
relations , and let for

a) Write in terms of

b)Show that
c)Show that .

Ans: a)

b)
We have,

c)Let

Now,

So

So .

76. Let be the set of all sequences of non-negative integers such


that

(i) ; and

(ii) there exists a positive integer such that for all .

Define the dual of the sequence belonging to to be the sequence


, where, for and is the number of 's which are greater
than or equal to .

(i) Show that the dual of a sequence in belongs to .

(ii) Show that the dual of the dual of a sequence in is the original sequence
itself.

(iii) Show that the duals of the distinct sequences in are distinct.
Ans: Since number of 's which greater than is number which greater
than
So for all
And since number of 's which is
This give us belongs to

Since number of which is equal to


So number of which is equal to
So dual of dual of sequence is original sequence

Let is function that


We have so is injective
Hence the duals of the distinct sequences in are distinct.

77. Find the value of

Ans:

78. If is a real-valued function of a real variable such that


for all find the function
Ans: We have,
for all real
Substituting by we get
for all real
Solving these two equations, we get,
and also

79. Solve

Ans:

Putting we get,

or
Now,

or
Similarly,

or
Hence solutions are

80. If is an odd positive integer, prove that for any ,

is divisible by .
Ans: Let again

Adding we get,

Now, for odd ,


, , ......., that is,
divides each term of .
So
Again for odd
, , ,......,
,

So

But are coprime. So

Surely is even and is also so.

Hence .
81. Find the number of rational numbers , where are relatively prime
positive integers satisfying and .
Ans: The prime factors of are . So there are total
prime divisors of

So number of ways can be resolved in two relatively prime factors

Among the integer one is smaller, other bigger. Hence they can be expressed
in form , and

So .

82. Let be a sequence

for . Find an expression for .

Ans: Putting ,we will get

Putting ,we will get

Putting ,we will get

So clearly we can see,that

So let us claim that .


then we will use this to prove the given equation.
83. Show that,

.
Ans: for

Now,
Using relation,

Adding we get,

Similarly using second relation we can achieve,


Combining the results we get,

84. Let where


are real numbers. We write , if for some , ,
but . Show that for
and , if and
, then .
Ans: and

Since and ,we have for all and for

Hence we have, .

85. Show that if is any odd integer greater than then is divisible
by
Ans: For odd we have,
[See Tomato subjective ]

If then we have [Since ]

Now if then,

Hence
86. Let { }.Find the number of unordered pairs { } of subsets
of such that and are disjoint,where or or both may be empty.
Ans: You can simply pick, for each element, if it goes in A, in B, or in
neither, for ordered pairs of subsets.
Each ordered pair except for is counted twice so the number of

unordered pairs is .

87. Consider the set $S$ of all integers between and including $100$ and
$99999$.Call two integers $x$ and $y$ in $S$ to be in same equivalence
class if the digits appearing in $x$ and $y$ are the same.For example ,if $x =
1010,y = 1000$ and $z = 1201$, then $x$ and $y$ are in same equivalence
class, but $y$ and $z$ are not.Find the number of distinct equivalence classes
that can be formed out of $S$.
Ans: For one digit classes, there are classes, each for all numbers formed by
each digit excluding .

For two digit classes, there are classes for the same reason.

For three digit classes, there are classes;

For four digit classes, there are classes, and four five digit

classes, there are classes. So, in total, there are


classes.
88. Suppose that one moves along the points in the plane where and
are integers in such a way that each move in a diagonal step,that is,consists
of one unit to the right or left followed by one unit either up or down.

(a) What points can be reached from the origin?


(b)What is the maximum number of moves needed to reach such a point
?
Ans: The invariant is . So, from the origin, we will reach all
points , where , and either or . That is
coordinates with same parity can be reached.

For the maximum, assume that the point is , where . (Otherwise, the
point symmetric to the line will have the same number of moves
required.) We can reach the y-coordinate in moves; all these moves should
be to the same direction as the line in the quadrant goes.
This way, we need another moves to reach the x-coordinate . These
moves should be in a zigzag pattern to ensure that we stay in the same y-
coordinate. So in total, we need moves. Since we assumed that
is the larger of the two values, our answer would be .

89. Let where are complex numbers.

If is real for all real numbers ,show that are real numbers.

In addition to above , assume that is not real whenever is not


real.Show that .
Ans:
If is real for all real numbers ,show that are real numbers.
[/quote]
is real is real. is real is real
is real and hence are real.

Suppose assume . Then, consider . Let it have


roots . If are imaginary, then must be imaginary which is

false. So, are real. Now, using the fact that , we have

coefficient of in and so, is

real while is imaginary which contradicts our assumption that


.

90. A partition of a set is formed by disjoint nonempty subsets of whose


union is .For example {{ },{ }{ }} is a partition of the set {
} consisting of subsets { },{ } and { }. However {{
}{ }} is not a partition of .

If there are nonempty subsets in a partition,then it called a partition into


classes.let stand for the number of different partitions of a set with
elements into classes.

(i) find
(ii) Show that .
Ans: 2)
see partitioning elements in can happen in two ways.

let the elements are

then either case 1: form k partitions which can be done in


ways and then the remaining element choose one of these k-partitions (so, k
ways) and go to it.

so under case 1 different possibilities.

case 2 : form k partitions(which can be done in ways)and


the remaimng element forms a new partition which consists of only.

so under case 2 different possibilities.

so

1) using 2) we get

91. For a finite set ,let denote the number of elements in the set

(a)Let F be the set of all functions


{ ... } { ,..., }
satisfying for every
Show that

(b)Let denote the number of functions in satisfying


For prove that

Using prove that for


.
Ans: a) no of options in first position is and for 2nd position ,3rd
position , so on.
so total no of functions are .

(b) for any function in either case 1 or case 2

case 1 : no of functions under this case


case 2: here

so,

so here the total no of functions are

so,

(c) using (b)

92. Let { } be a set of elements called voters. Let {


improper subset of } be the set of all subsets of . members of are
called coalitions. Let f be a function from to . A coalition being an
improper subset of is said to be winning if ; it is said to be losing
coalition if Such a function f is called a voting game if the
following conditions hold:

(a) is a winning coalition.

(b) the empty set is a losing coalition.

(c)if is a winning coalition and , then is also winning.

(d)if both and are winning coalitions , then and have a common
voter.

Show that the maximum number of winning coalitions of a voting game is


. Also find a voting game for which the number of winning coalitions is
.
Ans: We have, every subset of has a complement such that

Clearly, if belongs to a winning coalition, belongs to a losing coalition.


But the converse is not necessarily true. So to maximise winning coalitions,
we always put in winning side and in losing side so that there is a
bijection between these two partitioned sets.

Since the number of elements in winning and losing side is same( say)
this way, and number of subsets of , we have,
To construct a voting game with winning coalition we do the
following:

*Put { } in winning side.


*By condition , { },{ }............,{ } are the two element winning
subsets of
{ },{ },......,{ },{ },....,{ }.........., are the three element
subsets,........, is the subset of winning side.
*All the complements of above are losing.

We observe that, the total number of winning coalitions is the sum of -


combinations, combinations, combinations,..........., combinations of
the elements in excepting

So, number of winning coalitions=

93. If is a real number with , then show that the values taken by the

function , as varies over real numbers, range over all real


numbers.
Ans: Cross multipliying and simplifying the given expression, we get,
Now since is real, discriminant of
above expression

The above is satisfied iff this quadractic in has discriminant


implying,
which is the given condition,
hence will always be real.
94. In the adjoining figure, is perpendicular to , and the ratio of
lengths to is . The angle is and angle is . Find
expression of angle in terms of and .
Attachments:

In the adjoining figure, is perpendicular to , and the ratio of lengths


to is . The angle is and angle is . Find expression of
angle in terms of and .
Ans: Applying mn theorem (Some also call projection theorem) of
trigonometry,

So,

95. A man walking towards a building, on which a flagstaff is fixed


vertically, observes the angle subtended by the flagstaff to be greatest when
he is at a distance from the building, If be the observed greatest angle,
show that the length of the flagstaff is .
Ans: Let be the flagstaff(see above figure). Then maximum angle
subtended will be at where circle through and has as tangent.

Let then [angle in alternate segment]

So,
[from tr. , ]

So, .

96. Let be a cyclic quadrilateral with lengths

and .Show that .


Ans: ( , , , )=( , , , ) and ( , )=( , )

Likewise (by 2nd Cosine rule),


Dividing and taking square root, or

97. Prove by induction or otherwise that

for every integer


Ans: Let us assume,

Clearly, for is true.


Now we assume that is true for
Hence,

Also we have,

Hence is also true. Thus by induction hypothesis, we can say that


is true for all intger

98. Find the maximum among


Ans: Suppose,
Hence,

Thus after the function is decreasing in nature. Hence


Now we have to determine the greatest among
Clearly,
Also,
Hence is the greatest among all the listed real numbers.

99. A pair of complex numbers is said to have the property if for


every complex number we find real numbers and such that
.Show that a pair of complex numbers has property if and only if the
points and on complex plane are not collinear.
Ans: Let us assume the contrary. So are collinear and has same
argument .
So

So

So is collinear to .
But this way, the complex numbers not collinear to them cannot be
represented .So contradiction arises.

Hence, a pair of complex numbers has property if and only if the points
and on complex plane are not collinear.
100. Let . Consider the region, including the boundary,
enclosed by and . Find the maximum value of
in this region.
Ans: To maximise,
in the region

Taking equations, and solving we get


Taking equations, and solving we get
Taking equations , we get

So corner points are

At
At
At

At both value of is more than that the value of .

So maximum value of in the given region is .

101. Let and be two fixed points cm apart.


a) Let be any point non-collinear with and , such that . The
tangent at to the circle passing through the points and meets the
extended line at point . Find the length of the segments and .
b) Hence or otherwise, prove that the locus of all points in the plane such
that PA=2PB is a circle.
Ans: Let So angle [angle in alternate segment]

In triangle ,

In triangle
Now [power of point]

So

b)Let us assume be required point and for convenience that


coordinates of are respectively.By condition,

Above is the locus of a circle. So locus of is circle.

102. Let be a non zero complex number such that . Let be the
point in the complex plane and let be the point Let be the circle {
} and let be the circle passing through and . Show that and
intersect orthogonally.
Ans: Let the circles intersect at . We see that, and are on same
straight line since argument of are same. Let , then

So

So by power of a point, is tangent to . Also it is radius of , So and


intersect orthogonally.
103. In a certain game, balls of different colours are kept inside a sealed
box .You are told only the value of but not the number of balls of each
colour . Based on this, you have to guess whether it is possible to split the
balls into groups of each, such that in each group the three balls are of
different colours.Your answer is to be a simple or .You win or loss
a point according as your guess correct or not.For what values of ,you can
say and be sure of winning?For what values of , you can say and
be sure of winning?Justify your solution.
Ans: Solution: No & sure of winning :- .

Justification: If colours are there the balls cannot be of different colours &
if then there exists a possibility of splitting into two groups of each in
each group balls are of different colours.

For take ball of each colour & the splitting is possible.

Solution: Yes & sure of winning

Justification: By we can say that number of ball of a certain colour is at most


two.So we can keep them in different boxes.For it is not possible .

For example take balls of different colours & ball of


different colour.Now by ball of same colour are placed in two
groups.So at least one group contain more than one ball of same colours.

104. Let be a polynomial with integer coefficients. Suppose that there


exist distinct integers , such that
. Show that there does not exist any
integer with .
Ans: Consider the auxiliary polynomial . Clearly, according to
the problem, has four distinct integer roots . Hence we may
write
where is an integer
coefficient polynomial. (Since by factor theorem if is a root of a
polynomial then is it’s factor).

Suppose there exists an integer such that ; then


.Hence . Since are distinct, so
are . Therefore the equation indicates that can be written as the
product of at least different integers which is impossible (as is a prime).
Indeed even if we consider and , we have only three integers whose
product is .

Thus we have a contradiction and proving that there does not exist an integer
such that .

105. Tangents are drawn to a given circle from a point on a given straight
line, which does not meet the given circle. Prove that the locus of mid-point
of chord joining the two points of contact of the tangents with the circle is a
circle.
Ans: In the adjoining figure, we shall try to show that if the point varies in
the straight line, the position of remains unchanged.

From similarities of and we get,

Now from similarities of and we get,

Clearly is the radius of the circle and is the distance of the line from
the center of the circle. Hence and are fixed. Thus the point is
fixed. Now as the point moves in the straight line, the point moves
maintaining where and are fixed. Hence the locus of is a
circle with fixed diameter

106. How many words can be formed using the letters and
so that each letter appears at least once in the word?
Ans: Number of ways words can be formed using letters so
that each letter appears at least once in the word
= Number of ways word formed using – Number of ways
letter word formed using any two of + Number of words
formed by using . [ using inclusion exclusion principle. ]

107. Consider the system of equations . Find conditions


on and under which
(i) the system has exactly one solution;
(ii) the system has no solution;
(iii) the system has more than one solution.
Ans:
(i) the system has exactly one solution if two lines intersect each other, that
is
(ii) no solution if the lines are parallel, that is &
(iii) the system has more than one solution if the two lines are same, that is
& .

108. If any one pair among the straight lines


intersect, then show that the
three straight lines are concurrent.
Ans: Three lines are concurrent if each of them is linear combination of other
two & they are not parallel.
Now given one pair intersect that is they are not parallel.
Now

So they are concurrent.

109. Find the vertices of the two right angles triangles, each having area
and such that the point lies on the hypotenuse, and the other two sides
are formed by the and axes.

Ans: Suppose the vertices are and . Clearly or ab = 36.

Also the equation of the line through is . Since we

know that is on that line, there fore .

In this equation, lets replace by . Hence we get .


.
Therefore we get a quadratic in .
Thus or implying or
.
The other two vertices are: and OR and .
110. Let be the function defined by , and
, for , where is the set of all non negative
integers. Prove the following results:

i) for all
ii)There exists precisely four non-negative integers for which
.
iii) is divisible by , for all .
Ans: Clearly , and . Hence is positive. We
use strong form of induction and assume that up to , all is positive.

Then .

Since is positive. Hence . This solves first part.

For second part we observe

As we have .
Next observe that

So as
So we got four numbers , for which by observation.
We wish to show that this does not happen for any value of n greater than .

Clearly, if , then . We will show that for all


greater than . (If we can show that then it will automatically show that
is not equal for any value greater than , implying not equal to
for any other value apart from the ones that we have found earlier by trial
and error).

Note that . That is . Similarly


implying .
We will again use strong for of induction. Suppose for all values of
from to n . Using this assumption we will show that f(n+1) > n+1.

. By induction and . This


implies

But 2n-1 > n+1 (as n > 2). Hence it follows that f(n+1) > n+1. This solves
second part.

Finally we wish to show that divides .

We again use induction. First note that hence divisible by . Assume


that the claim is true for . That is divides . Using this
assumption we wish to show that divides . Now note that:

Clearly is divisible by . Also is divisible by as by


inductive assumption is divisible by Hence their sum
is divisible by . This shows divides divides

This completes the proof of third part.


111. All the permutation of the letters are written down and
arranged in alphabetical order as in dictionary. Thus the arrangement is
in first position and is in second position. What is the position of the
word ?
Ans: Number of arrangements with:

* at first=
* at first=
* at first=

Now, Number of arrangements with:


* first, second=
* first, second=
* first, second=

Now, number of words before with first, second is two, namely


and .

So total words before

So position of

112. Show that a necessary and sufficient condition for the line
where are non-zero real numbers, to pass through
quadrant is either or .
Ans:

Now the given line pass though the origin iff either of the intercepts are
positive,i.e.,
either or
i.e., either 0r [ ]

113. Consider the function,

i) Is continuous at ? Justify.
ii)Show that does not vanish in the interval , and indicate the
points where changes its sign.
Ans: we have when , and when , if
so,

LHL ,

RHL

hence discontinuous.

for second part, intermediate value theorem is applicable iff function is


continuous. So does not vanish in the interval though it
changes sign at neighbourhood of .

114. A function is called periodic if for some constant ,


for every real number . Show that the function

is not periodic.
Ans: Let us assume

In a particular case, when ,

Which is possible iff

or, for some { }

So

Which is contradictory since RHS of above is rational whereas LHS is


irrational.

So claim of question is proved.

115. Show that for all positive integer , divides


Ans: Let

Clearly

Let divide for . Then for some positive integer

So

Clearly above is divisible by .


Hence by induction.

116. Prove that is divisible by for all


Ans:

Clearly, above is divisible by .

117. How many numbers less than are there, sum of whose digits equals
?
Ans: Required solution= coefficient of in

which is

118. i)Find the number of all possible ordered of non-negative

integers such that .


ii) Show that the number of possible ordered of non-negative
integers such that
is
Ans: i) Required number =number of non-negative integral solution of

ii)Required number=Number of non-negative integral solutions of

+Number of non-negative integral solutions of +Number of non-

negative integral solutions of +..........................+Number of non-

negative integral solutions of

...................................
....................................
119. For a real number , let denote the largest integer less than or equal
to , and denote . Find all solutions of the equation

Ans: Let , where ,i.e, the integral part and is the fractional
part, .

Since and are integers so is surely integer.

Let

So

The only integers such that and is integer, are,

So required solutions are and

So .\

120. Let the circles


and with centres at and
intersect at and . Show that the points and lie on a
circle.
Ans: Let be one of the intersecting point.
Centre of first circle is , centre of second circle,is

We know, angle between radii of circles at point of intersection

So
Above is the locus of circles .
It obviously passes through since is another intersecting point. Also it
passes through

It is easily checked that, and satisfies the above locus.

Hence,the points and lie on a circle.

121. i)In the identity,

Prove that
ii)Deduce that
Ans: i)We have,

Putting, we get

.............
...........

This way,
ii)Putting ,

Putting

gives,
122. Let where and are successive positive integers and
Prove that is an odd positive integer.
Ans:

Hence,
Clearly, is even. Hence is odd

123. Show that

Ans: The identity is true for


Let it be true for

Adding to both sides we get,

So identity holds for


Hence by induction the identity is proved.

124. Let for


Show that for each is an integer.
Show that for all
Use above, to show that is divisible by
Ans: For simplicity, we will first prove

From the expression of we see that and are integers. Hence by


that we have proved, we can say that is an integer for all

Lastly we use induction to prove


We assume that and
Hence and
Then,
Thus by induction, we can say that is divisible by

125. Each pair in a group of persons is classified by the existence of


kinship relation and friendship relation between them. The following table of
data is obtained from such a classification.
Determine(with justifications) whether each of the following statements is
supported by the following data:
(i)Most of the friends are kin.
(ii) Most of the kin are friends.

Ans: i) From above, pairs of people are both friends and kin, but only three
pairs of people are friends but not kin.
So statement (i), i.e., most frinds are kin,holds.

ii) pairs of people are both kin and friends but pairs are kin but not
friends.
So statement (ii),i.e., most kin are friends, is false.

126. Find the different number of ways different gifts can be presented to
children so that each child receives at least one gift.
Ans: i) If the gift distribution is

Then total ways of distributing=


ii) If the gift distribution is

The two children getting two gifts can be selected in ways.

So total ways of distributing gifts=


So total ways of distribution=

127. We say that a sequence { } has a property , if there exists a positive


integer such that for every . For each of the following
sequences, determine whether it has the property or not.[ Do not use any
result on limits]

i) ii)
Ans: i) It has property
Clearly and for all

a) are all lesser than .


b) are are lesser than where are the even integers

ii) It does not have property


For every [[aopsnowrap =[/aopsnowrap] a positive integer]

Hence
Since there are infinitely many such , so there is no such that
i) It has property
Clearly and for all

a) are all lesser than .


b) are are lesser than where are the even integers

ii) It does not have property


For every [ a positive integer]

Hence
Since there are infinitely many such , so there is no such that

128. For any integer , let be the integer nearest to .


a)Given a positive integer , describe all positive integers such that
.
b)Show that

Ans: a) We have,

But since are integral, we have,

b)

129. Let be the line segment of fixed length with its its two ends sliding
along the and respectively. Complete the rectangle where
is the origin. Show that the locus of foot of perpendicular from to is
given by
.
Ans: Let coordinates of be , then is , is

Equation of is

Let be foot of perpendicular from to .


Since it is perpendicular to and passes through its equation is,

Solving we have

Evaluating and adding, we get,

Which is the locus of an asteroid.

130. Out of a circular sheet of paper of radius , a sector with central angle
is cut out and folded in shape of a conical funnel. Show that the volume of

the funnel is maximum when equals


Ans: Arc length of sector=

if radius of base then, or

Slant height of cone =radius of sector=

so height is given by

Volume
Let us maximise where

So

It can be easily checked that is negative at

Hence,the volume of the funnel is maximum when equals .

131. A regular five-pointed star is inscribed in a circle of radius .(See the

figure). Show that the area of the region inside the star is
Ans:
, see diagram.

Rule of sines in the .


.
.

Area .
Area of the region inside the star:

.
132. Using the identity , , or otherwise, prove that

.
Ans: Let and
So is continuous and differentiable everywhere.

So by Lagrange's MVT, there exists at least one value of , say , such


that,

Now
Now putting [Since ] we get,

133. a)Study the derivatives of the function

to make conclusions about the behaviour of the function


as ranges over all possible values for which above formula for is
meaningful.
b)Use information obtained in (a) to draw a rough sketch of graph of on
plain paper.
Ans:

in and otherwise
So increases in , decreases in ,

Hence is a point of minimum, is point of maximum.

Maximum of =

Minimum of =

Also has infinite discontinuity at


134. Sketch on plain paper, the graph of the function in the range

Ans:
is a even function, and it oscillates faster than as
increases more rapidly than
So the graph is:
135. Let be a fixed odd positive integer. Find the minimum value of
, where are non negative integers and
Ans: Let then . Clearly
decreases on left and increases on right of . So the function is
minimum at
But is integer and a fraction( is odd). So we must check values of
at integers nearest to in its left and right which are and
respectively.

Clearly

Hence required minimum value is

136. In a competition, six teams play each other in the


preliminary round- called tournament. Each game end in either a
win or a loss. The winner is awarded two points while the loser is awarded
zero points. After the round robin tournament, the three teams with the
highest scores move to the final round. Based on the following information,
find the score of each team at the end of the round robin tournament.
i) In a game between and , team won.
ii)After each team has played four games, team has points, team had
points and team had
four points. The remaining matches yet to be played were
i) between and ;
ii) between and ; and
iii) between and
iii) The teams and had won their games against and
respectively.
iv) Teams and had moved to the final round of the tournament.
Ans: i) Let us first consider , till four rounds, it had got points, hence won
all his four matches against . But it lost to in its last game.
So final score of is

ii)Now consider , till four rounds it had points, which means he lost one
game till then. But in last round it lost to and previously we saw that, has
beaten .
So score of is

iii) Till four rounds, had points, i.e, it won two, and according to previous
observations, it lost to , and won against . Finally it lost to .
So score of is

iv) had moved to final, but it had surely lost to and won with . But to
make it to top three it must surpass points of and achieve at least points.
So it won against .
So score of is

v) lost to and beaten from previous observations.


So score of is
vi)We see that won only with .
Score of is

So the scores are

137. Find the area of the region in plane, bounded by the graphs of
and
Ans: Clearly the blue, red and green graphs depicts
respectively.

and intersects at on first quadrant. and


intersects at . and intersects at .

So the corner points whose area are to be found are .

So the area is | |
square units.
138. Show that the area of the bounded region enclosed between the curves

and , is .
Ans: learly red and blue graphs are of and respectively.
Clearly since the graphs are symmetric about ,
area enclosed by the graphs in first quadrant=area enclosed by them in
second quadrant

Clearly the graphs intersect at in first quadrant, and cuts


at

So required area
Attachments:
139. a)Given identical symbols, say , show that, the number of boxes
you can distribute them in boxes marked so that no box goes empty

is .
b)In an arrangement of and , an uninterrupted sequence of one
kind of symbol is called a . (For example, the arrangement
of and opens with ab of length
, followed successively with a of length , an of length , a
of length and finally, a of length .Find the arrangements
of and in which there are [You may use (a)
above].
Ans: a) Let the boxes be denoted by variables
Then the requires number is the number of positiove integral solutions of
i.e, coefficient of in , i.e.,

b)Clearly, by (a), the can be arranged in ways.


But in between , there are gaps where the can be placed.
Also considering the extremities, there are two more places where the can
be placed.
So there can be or runs of .
So arrangement of

So required total arrangements

140. Prove that:-

where is the greatest


positive integer such that .
Ans: Let be the required summation.This problem requires attention,let's

take the cube roots of unity, .Now we have .Also

observe that and .Now


add up these equations and use the relation to get
.
Now

So
Apply De Moivre's theorem,

Hence

or
Alternatively .Taking modulus and using triangle
inequality, .

141. Show that the larger of the two areas into which the circle

is divided by the curve is


Ans: Clearly the blue and red graphs are the upper half of and
respectively.
Since both curves are symmetric about , it is sufficient to calculate
the larger area of upper part and multiply by .

Clearly cuts at and cuts at

The smaller area between the curves


Area of the circle with radius is

So required area=
Attachments:

142. A cow is grazing with a rope around its neck and the other end of the
rope is tied to a pole. The length of the rope is metres. There are two
boundary walls perpendicular to each other, one at a distance of metres to
the east of the pole and another at a distance of metres to the north of the
pole. Find the area the cow can graze on.
Ans: We have a circle with centre and radius m. Let and be the
boundary walls present. The striped region shows the area available for
grazing. Now if we see carefully we can see that area of the shaded region is
nothing but the sum of the areas of the rectangle AEPC, the two right angled
triangles and and the sector .
radius of the circle =

AS (Pythagorean property of a right angled triangle)

From here, we get

Similarly, as

(pythagorean property)

Therefore,

Area of rectangle

Area of triangle

Area of triangle

Area of sector

Now ,

Therefore area of sector

Thus Total area of the Grazing Field


143. Draw the graph(on plain paper) of
min{ }
Ans: *Graph of is same as, , where is non negative. For
negative values of ,it is the graph of reflected about .

*Now we apply transformations to find the remaining graphs.

*Graph of can be found by lowering the graph of by unit


along

*Graph of can be found by first shift the graph of along


positive direction of by unit, and then lowering it by unit along
.

*Graph of can be found by first shift the graph of along


positive direction of by unit, and then lowering it by unit along
.

*Now we will plot all the graphs together and then consider the portion
which are ‘lowest’.
(all of them together)
(considering only the minimum portions)
Graph of

144. Sketch, on plain paper, the graph of


Ans: 1.Domain: The function is defined at all real numbers except and

2.Even/Odd: Clearly . Hence it is sufficient to investigate the


function for positive values of and then reflect it about .

3.Critical Points: Critical Points are those values of for which the first
derivative of is either or undefined. here

Hence critical points are


4.Monotonicity: The first derivative is negative for all positive values of
(note that we are only investigating for positive values, since we can then
reflect the picture about as previously found). Hence the function is
‘decreasing’ for all positive value of .
5.Second Derivative: We compute the second derivative to understand a
couple things:
convexity/concavity of the function
examine whether the critical points are maxima, minima, inflection points.

So at we have local maxima. Since , we have as a local


maxima.
Also the second derivative is negative from to and positive after
.Hence the curve is under-tangent (concave) from to , and
above-tangent (convex) from onward.

6.Vertical Asymptote: We next examine what happens near . We want


to know what happens when we approach from left and from right. To
that end we compute the following limits:

7.Horizontal Asymptote: Finally we examine what happens when x


approaches . To that end we compute the following:

Using above informations we draw the following graph:


145. Find the number of positive integers such that none of them is
divisible by , , .
Ans: Let denote the occurence of a multiple of , denote that of and
denote that of .Clearly the required number of cases= =
.Now
=no. of multiples of +no. of
multiples of +no. of multiples of -no. of multiples of -no. of multiples of
-no. of multiples of +no. of multiples of .

146. Use the derivatives and left and right limits at the points of
discontinuities , if any, of the function
to make conclusions of the behaviour of the function as
ranges over all possible values. Using this, draw a rough sketch of the
graph of function on plain paper.
Ans:

and

So is maximum at . increases on left and decreases on right


of
Maximum value of is
Clearly is undefined at , hence discontionuous there.

Let

Equating highest powers of (i.e.coefficient of ,i.e, ) and (i.e.


coefficient of ,i,e,, ) to we get
So asymptotes are . Since equation is two degree, it has no more
asymptotes.

Using above, we trace the curve:


147. For the following function study its derivatives and use them to sketch
its graph on plain on plain paper.

for
Ans:

and So is a point of maximum.


increases on left and decreases on right of
Clearly is undefined, hence discontinuous at

Using above, we trace the curve:


148. Study the derivative of the function
and roughly sketch the graph of on plain paper.
Ans:

Clearly when or and when


So decreases strictly in and increases strictly in and

So is maximum at , minimum at .

So maximum value of is
Minimum of is

So the graph is as follows:


Attachments:
149. Show that for every positive integer , is either an integer or an
irrational number.

Ans: Let us assume where

So or , putting for integral ,


we get,

But it is contradictory that divides both as


So either or is not rational and expressible in form.
I.e. is either an integer or an irrational number.

150. Suppose that the roots of are rationals and are


integers.Prove that the roots are integers.

Ans: Let the roots be and


By rational root theorem we have and
Hence implying the roots to be integers.

151. Let { }. For in , we define to be the


remainder obtained by dividing the product by . For example,
and . Let be an element in . An element in is
called the inverse in if . Find which of the elements
have inverses and write down their inverses.
Ans: Clearly to find the inverse of a element we need to solve for the
equation
i.e, where is the usual multiplication.

For it is impossible to have inverses as they are even and


cannot have solution in .
The smallest possible solution of is . For other elements in , has
no inverse belonging to . Hence inverse of is unique.
In a similar manner, it can be shown that are inverses of .
Finally does not have inverse as no element in is divisible by .

Hence have inverses which are respectively.

152. Sketch the curve on plain paper and show that the
equation (in )
(where the real constant is such that ) has three
distinct real roots all of which has absolute values smaller than .
Ans:

Clearly increases when or and decreases


when . So is maximum at minimum at .

Using above we trace the curve of .

is negative, increases in hence intersects once in


that interval. Again, it assumes maximum value at and
thereafter decreases upto where it assumes minima . So
it has a root at . Lastly and increases strictly in
. So again has a root in .

So all three roots of has absolute values less than .

the required graph:


153. Sketch the set in the Argand plane, where
and
Ans:
where and are real.
Then for set , we have

For set we've, which is the region

within a parabola wit its vertex at and the real axis as its axis.

Hence denotes the left half of the region within the aforesaid parabola,
that is the region lying in the second and third quadrants.

154. Let denote the largest integer less than or equal to . For example,

. Draw a rough sketch of the graphs of the following on plain


paper:
(i)
(ii)

(iii)
Ans: i) Clearly , where .

ii) { } where { }= fractional part of . Clearly { }

iii)
and so on.

So the graphs are, respectively:


155. Draw the region of points in the plane, which satisfy .
Ans: For tracing the region, first we draw , and reflect it about
to get the graph of , then we shade the region sandwiched between
the two lines and and containing , in the range to
get the required region.

156. Sketch, on plain paper, the regions represented on the plane by the
following:
i)
ii)
Ans: i) For transforming to we omit the portions where
sine curve is negative, and trace both the original sine curve and its reflection
about where sine curve is positive.
ii)Firsy we transform to by erasing the curve for
negative values of and reflecting the curve for positive values of about
. Then we transform to by omitting the
portions of where is negative,and tracing both the original curve
and its reflection about where is positive. Lastly we see
that, (0,0) doesn't lie in the region so we shade the regions, which
does not contain the origin.

The graphs are as follows:

157. Find all positive integers such that, , where, for any
real number , is the greatest integer less than or equal to .
Ans: We trace the curves of and .
Clearly integer values of where the curves intersect are
hence they are required
integral solutions.

158. Using Calculus,sketch the graph on plain paper of the function


for
Show that the function defined as above for numbers attains a
unique minimum.What is the minimum value of function ? What is the value
of at which minimum is attained ?
Ans:

. Since is positive, only root of


is
Clearly for all positive in left of and for all in
right of .
Hence decreases in left of , reaches minimum at and increases
thereafter.
Since changes sign only at so has unique minimum.

Minimum value of is

So required graph is:


159. Let denote the largest integer (positive, negative or zero) less than or
equal to . Let be defined for all real numbers .

(i) Sketch on plain paper, the graph of the function in the range
(ii) Show that, given any real number , there is a real number such that

Ans: i)Note that graph of is same as of , which is the


positive portion of the parabola in the domain .

Clearly part only increments (or decrements) it by integer quantity as is


constant between any two integers. That for any integer for all

Hence graph of is given below.


ii)Finally consider and arbitrary value . We take
f(x_0)= [x_0]+\sqrt{x_0-[x_0]}=[y_0]+\sqrt{(y-[y_0])^2}=y_0(since
)

Alternatively, since it it clear from the graph that, every horizontal line in the
plane cuts the graph of at least once, hence is invertible, i.e., given
any real number , there is a real number such that
160. If are roots of the equation , find the

equation whose roots are .


Ans: We have by Vieta's Theorem,

So, Similar for,


Let

Hence the required equation is obtained by putting in place of in the


given equation.

The transformed equation is


161. By considering the expression where is a
positive integer , show that the integers are alternatively even and
odd as takes values Here for any real number , denotes the
greatest integer less than or equal to
Ans: Let
which is even.
Let and be even integers. Let where

Clearly above is even so by induction, is always even.

Let

When
So .Clearly
So,
which is odd.

When
Clearly
.
So which is even.

Hence, the integers are alternatively even and odd as takes


values

162. Suppose that is an arc of a circle with a given radius and centre
subtending an angle is fixed . Consider an arbitrary point on
this arc and the product , where and denote the
lengths of the straight lines and , respectively. Determine possible
locations of for this this product will be maximised. Justify your answer.
Ans: Let arc subtend angle at centre. So arc subtend angle at
centre.
Let be the centre of the circle.

In triangle [say] So
Similarly in triangle for some constant .

Let

, [since are positive as


]
So is maximum at . that is subtends exactly equal angles at
centre and hence is the mid point of arc .

163. i) If where is a positive integer, show that


ii) Let triangles and
be inscribed in the same circle. If the triangles are of equal perimeter,
then prove that
iii)State and prove the converse
of (ii) above.
Ans:
i)

When is odd,

[as is
odd]
When is even,

[as
is even]
Combining both cases,
ii)Let and be sides of and respectively, and be the
radius of circle.
Given

[by applying sine rule]

iii)Conversely if

i.e, perimeters are equal.

164. In a club of members, members play none of the games Tennis,


Badminton and Cricket. members play exactly one of the three games and
members play exactly two of the three games. members plat at least one
of the games among Tennis and Badminton, whereas members play both
Tennis and Badminton. Determine the number of Cricket playing members.
Ans: Number of playing members=
Number of players playing exactly all three games=

play both tennis, badminton, among them play all three games.
Hence, play only tennis and badminton.

So members play exacty two games with cricket between one of


the games.
Now play one of the combination of games:(cricket,tennis),
(cricket,badmintion),(tennis),(badmintion)
So players playing only tennis or only badminton=
players play only one game; so plyers playing only cricket=

So play only cricket, play all games, play exactly two games with
one of them being cricket.

So total cricket playing members=

165. Find all such that


and
Ans:

So the cases are


i) but so
So

Plugging this in we get


. So the solutions for this case are

ii) So
[since |y|\geq 0]
So Solutions for this case are .

iii) , solutions for this case are

So the complete set of solutions is { }


166. i) Determine so that the equation
has four real roots in arithmetic progression.
ii)Let and be two real numbers. If the roots of the equation
have absolute values less than one, show that each of the
following conditions holds:
(i) (ii) and (iii) .
Ans: i)Let the roots be
by Vieta's rule i.e, roots are
Product of roots taking two at a time

product of roots=

Solving we get

ii)Let be the roots.


so so but

Let us contradictorily assume


But since
so hence a contradiction arises, i.e,

Let us contradictorily assume that


but so
so hence contradiction arises,i.e.,

So all the three given conditions hold.

167. A rectangle with the two axes as two sides, the origin as a
vertex is drawn in which the length of is four times the width .A
circle is drawn passing through the points and and touching at its
mid-point, thus dividing the rectangle into three parts. Find the ratio of the
areas of these three parts.
Ans: Follow figure, the coordinates marked are
for some .
Equation of the circular part with centre is

It passes through
Let join of and intersect at (not in figure.)

So
Let

In triangle

So

Hence,

After calculations
168. An operation on a set is a mapping that associates with every pair of
elements and of the set , an unique element of . is said to be a
, if the following conditions hold:
i) for all elements and of ;
ii)there is an element if such that for all elements of
;and
iii) for each element of , there is an element of such that
.

If is the set whose elements are all subsets of a set , and is the operation
of defined as \ , show that is a group under .

(For any two subsets and of , \ denotes the set of all those
elements which are in but not in ).
Ans: i)
Let \ Clearly as per question, Venn diagram of
is as in figure i.
Its Venn Diagram is as in figure ii.

Let . Venn Diagram of is as in figure iii.


Its Venn Diagram is as in figure iv.
Clearly shaded areas of are identical. Hence,

ii)Let be the null set.Then

So \ \ \ , i.e, elements that


are not in but are there in ,i.e.,

So there is such that

iii)We Have
So \ \ , ie. elements that are in but does not
belong to , i.e,
So for all , there is such that .

So all conditions (i),(ii),(iii) are held.


Hence is a group under .

169. Show that if a prime number is divided by , then the remainder is


either prime or .
Ans: Let ,then we can see that if then remainder
if ,then ,now clearly ,since prime
factoristaion of so r cannot be a multiple of these numbers.
so remaining elements in the set of are the prime no.s between to and .
thus it is proved that if is a prime no. divided by then we get a remainder
either or a prime no. .

170. Consider the function

i)Sketch the graph of on


plain paper.
iii)Determine the point of discontinuities of and the points where is not
differentiable.
Ans: The nature of function repeats after units while starting from a fixed
point. Hence it has period Thus just analyzing in would complete
our task.
Suppose, in We have

Hence in

Hence in implying concavity in

Suppose, in We have

Hence in

Hence in implying concavity in

We also see that


Hence is continuous at

Again
Hence is differentiable at
It is easily seen that is discontinuous and thus obviously non
differentiable at
We conclude is discontinuous and non differentiable only at odd
integral points.

171. A function from set into set is a rule which assigns each element
in , a unique (one and only one) element (denoted by ) in . A function
of set from into is called an onto function, if for each element in
there is some element in , such that . Now suppose that {
} and { }. Determine the total number of onto functions
of into .
Ans: We know that the total number of functions from to are .
Now , number of functions with exactly two elements in the range is
and the number of functions with exactly one element in range is . So by
Principle of Inclusion and Exclusion , our answer turns out to be
...

172. If the coefficients of a quadratic equation are


all odd integers, show that the roots cannot be rational.
Ans: Suppose, if possible, we assume that the equation
has rational roots.
Clearly then the discriminant would be a perfect square of an integer. The
discriminant is
We notice that the discriminant is odd. Hence it is a perfect square of an odd
integer. We know that any odd perfect square can be expressed in the form
of where
Since is an odd integer, we can write,
where
Since are odd, is also odd. We can express in the form of
where
Hence,
where and
clearly
Contradiction!

173. i)Prove, from first principles, that,


for every positive integer .
ii)Prove that,
for every positive integer .
Ans: i) Here 1st principles is actually 1st principle of mathematical
induction.
For we have:

So De Moivre holds for . Assume now it’s true for . I.e.:


(1)
We want to show it is also true for , and then we’ll be done.
Consider
then:

.
We now use equation (1), and substitute in for
on the right hand
side. Then we get

after expanding the brackets.

Now, as we all know and


. So we have

Hence we are done by induction.

ii)From Euler’s formula, we know:

.
Taking both sides to the power gives us:

we get:

.
We use Euler’s formula again:

and therefore we have De Moivre’s Theorem:


.

174. If , , are the angles of a triangle, then show that

Ans: We are to show


Now,

and hence we are done.

Equality holds when and

175. Let and be real numbers such that the equations and
have exactly one solution. Then, show that the equations
and also have exactly one solution.
Ans:
Condition of to have one solution is

which is the condition for to have one solution.


Hence, the equations and also have exactly one
solution.

Geometrically, it is easy to see that and are perpendicular and and


are perpendicular. Since have exactly one solution, they intersect at
precisely one point, hence their perpendiculars,i.e., also intersect at
precisely one point, hence have only one solution.
176. Suppose and are positive integers greater than or equal to .Show
that is divisible by .
Ans: And here is my solution:-
We have:-

= since cancelling out terms from


gives .There are n terms on .Now,we multiply and divide by

.This gives:- = ,where is a


positive integer since the binomial coefficient is a positive integer.Thus
...which implies that is divisible by .

177. Let be an integer. Let be the largest integer which is less than or
equal to ,and which is a power of . Put the least common multiple of
. Show that, is odd, and that for every integer , ,
is even. Hence prove that

is not an integer.
Ans: Think of the sequence,

their LCM=product of the all primes which had occured in the prime
factorisations of numbers of above sequence, along with the highest
multiplicity with ehich they have occured
[ are positive integers]
where is odd.

So [by question LCM= , ]


or which is odd.
Again but
[ and is integral]

So is even.

All numbers in numerator are even except . Hence

So cannot be integral.

178. Suppose is a continuous function such that .Prove


that is identically equal to zero.
Ans:

differentiate to get so either as required or

integrating from to , or
but

we had

hence
hence putting this in above we have
179. Show that, number of ways one can choose a distinct positive integers,
each smaller than or equal to such that their sum is odd, is .
Ans: The given condition is possible if we choose an odd number of odd
integers, with any number of even integers, less than

Now odd number of odd integers less than can be chosen in


ways as there are odd numbers less than

Along with them,no even integer can be chosen in ways, one in


ways,..., in ways.

Required number=

[Proved]

180. Let be a polynomial


with integer coefficients, such that, and are odd integers. Show that:
does not have any even integer roots.
does not have any odd integer roots. Let
be a polynomial with
integer coefficients, such that, and are odd integers. Show that:
does not have any even integer roots.
does not have any odd integer roots.
Ans: a) Clearly, is odd. Now for even integer we have
is even. Also since is odd, we have
is odd even Hence even
does not have any even integer roots.

We have is odd and is odd. Thus


is even. For odd we know that
are odd. We also know that if we multiply an integer by
an odd integer, the parity of the original integer does not change. Hence
has the same parity as for Thus
has the same parity as
odd which is even. Hence is even for
all odd and therefore is odd. Hence it does not have any odd integer
roots.

181. If are positive numbers, show that

Ans: We have or or

so

applying similar inequalities for pairs and and adding and simplifying
we get

182. Find the set of all values of such that can take all real
values.
Ans:

Now takes all real values if discriminant is allways .

So now we have to find the all values of {m} such that for
all .

…(i)

Now this is a equation of upside open parabola. If the discriminant is of


equation (i) then will always positive.

Conclusion: If , then can take all the values as varies


over
183. Consider the function , .Let for each positive integer ,

be the polynomial such that for all .Show that

Ans:

Replace

184.Let be a differentiable function such that its derivative is a

continuous function. Moreover, assume that for all ,


Define a sequence of real numbers by :
Prove that there exists a positive real
number such that for all ,

Ans. Notice that

where
, and Hence,

Therefore,
Hence,
for . But we can verify that it also holds for
. Now, take . We obtain for
.This is exactly what we want to prove

185. Let be a continuous function such that for all and for
all , Show that is a constant function

Ans. First choose , then is constant on . Let to see that is


constant on . By continuity is constant on . In the same way, with
one shows that is constant on . Hence is constant.

186. Let . Let be an matrix such that


for all . Suppose that and

Show that is a multiple of .

Ans. First note that

Since the summands are so half of them should


be and half of them are This proves that is even. Say

So from
we can say that , since half ( ) of the summands are

Similarly, from we can say But

Hence we get that


which completes the proof.

187. Let be a real-valued continuous function which is


differentiable on and satisfies .Suppose there exists a constant
such that for all . Show that for
all

Ans.

Now, is
continuous in so there exists such that for all

This gives us

Again, Giving
Inductively, we get

Finally, from

we get, by letting that for all

188. Consider all permutations of the integers In how many of


these permutations will the number be the minimum of the first
numbers and the number be the minimum of the first 50 numbers?

Ans. Let us count all such permutations . We can choose freely


, and then for we are forced to choose the
minimum of the remaining images for , then again we can choose
freely and again we are forced to choose the minimum
of the remaining images for , then we can freely choose the remaining
images . This gives a total of

189. A urn contain red balls and black balls. A ball is drawn from urn and
it's colour is noted, returned to the urn.Further additional balls are added
to the urn of same colour. This process is continued. Define , if at the
ith drawn ball is red, otherwise.compute .

Ans. This is called Polya's Urn Scheme. Observe that


. Claim: .For , this is trivial. For ,
.If
this is true for some , then

will imply the same for .Now, using linearity of expectation,

190. Determine all for which the equation have


not solutions in Z.

Ans BY AM-GM

.
This implies

So all .

You might also like